Maximization

Algebra Level 2

Let a , b a,b and c c be positive real numbers satisfying a + b + c = 9 a+b+c= 9 . Find the maximum value of a b + a c + b c ab+ac+bc .


The answer is 27.

This section requires Javascript.
You are seeing this because something didn't load right. We suggest you, (a) try refreshing the page, (b) enabling javascript if it is disabled on your browser and, finally, (c) loading the non-javascript version of this page . We're sorry about the hassle.

4 solutions

Relevant wiki: Lagrange Multipliers

Target Function : f ( a , b , c ) = a b + b c + c a \displaystyle f(a,b,c) = ab+bc+ca subject to constraint a + b + c = 9 \displaystyle a+b+c=9 .

We have L f ( a , b , c , λ ) = a b + b c + c a λ ( a + b + c 9 ) \displaystyle Lf(a,b,c,\lambda) = ab+bc+ca-\lambda(a+b+c-9)

Evaluating the partial derivatives we get the four equations :

{ b + c = λ c + a = λ a + b = λ a + b + c = 9 \displaystyle \begin{cases} b+c=\lambda \\ c+a=\lambda \\ a+b=\lambda \\ a+b+c=9 \end{cases} , Solving we get a = b = c = 3 a=b=c=3 .

If we end it here it is incomplete, we must determine what the critical points leads us to, a maxima,minima or saddle point.

Taking the Hessian Matrix of L f ( a , b , c , λ ) Lf(a,b,c,\lambda) we see,

Δ = ( 0 1 1 1 1 0 1 1 1 1 0 1 1 1 1 0 ) \displaystyle\Delta = \begin{pmatrix} 0 & 1 & 1 & -1 \\ 1 &0 &1 &-1 \\ 1 & 1 & 0 & -1 \\ 1&1&1&0 \end{pmatrix} which is Negative definite matrix . Since Δ < 0 \displaystyle \Delta < 0 so f ( a , b , c ) f(a,b,c) attains it's maximum at ( a , b , c ) = ( 3 , 3 , 3 ) (a,b,c)=(3,3,3)

So, f max ( a , b , c ) = 27 f_{\text{max}}(a,b,c) = 27

Ohhhhhhhhhhhhhh God

Sayandeep Ghosh - 5 years ago

That's long :))

Steven Jim - 4 years ago
Harsh Shrivastava
May 23, 2016

Relevant wiki: Trivial Inequality

Firstly note that a 2 + b 2 + c 2 a b + b c + c a a^{2}+b^{2}+c^{2}\ge ab+bc+ca .

Since a + b + c = 9 a 2 + b 2 + c 2 = 81 2 ( a b + b c + c a ) a+b+c=9 \implies a^{2}+b^{2}+c^{2}=81-2(ab+bc+ca) .

Thus a 2 + b 2 + c 2 = 81 2 ( a b + b c + c a ) a b + b c + c a 27 a b + b c + c a a^{2}+b^{2}+c^{2}=81-2(ab+bc+ca)\ge ab+bc+ca \implies 27 \ge ab+bc+ca .

Lemma: a 2 + b 2 + c 2 a b + a c + b c ( ) a^2+b^2+c^2 ≥ ab+ac+bc \qquad (*)

Proofs:

There are many proofs of this inequality,I shall provide three of them.

Proof 1: Consider the following inequalities: a 2 + b 2 AM-GM 2 a b b 2 + c 2 AM-GM 2 b c c 2 + a 2 AM-GM 2 c a \begin{aligned} a^2+b^2 &\stackrel{\text{AM-GM}}\geq 2ab\\ b^2+c^2&\stackrel{\text{AM-GM}}\geq 2bc\\ c^2+a^2&\stackrel{\text{AM-GM}}\geq 2ca \end{aligned} Adding all these,we get: 2 ( a 2 + b 2 + c 2 ) 2 ( a b + a c + b c ) a 2 + b 2 + c 2 a b + a c + b c \begin{aligned} 2(a^2+b^2+c^2)≥ 2(ab+ac+bc)\\ \implies \boxed{a^2+b^2+c^2≥ab+ac+bc} \end{aligned}

Proof 2:

Since a , b , c a,b,c are positive reals,and because the inequality is symmetric,we can assume WLOG that a b c a≥b≥c .Then we have,by the Rearrangment Inequality: a × a + b × b + c × c a × b + b × c + c × a a 2 + b 2 + c 2 a b + a c + b c a\times a+b\times b+c\times c≥ a\times b+b\times c+c\times a\\ \implies \boxed{a^2+b^2+c^2≥ab+ac+bc}

Proof 3: Multiplying ( ) (*) by 2,we get: 2 a 2 + 2 b 2 + 2 c 2 2 a b + 2 b c + 2 a c ( a 2 2 a b + b 2 ) + ( b 2 2 b c + c 2 ) + ( c 2 2 a c + a 2 ) 0 ( a b ) 2 + ( b c ) 2 + ( c a ) 2 0 2a^2+2b^2+2c^2≥ 2ab+2bc+2ac\\ (a^2-2ab+b^2)+(b^2-2bc+c^2)+(c^2-2ac+a^2)≥0\\ (a-b)^2+(b-c)^2+(c-a)^2≥0 Which is true by the Trivial Inequality.


Returning to our original problem, notice that: ( a + b + c ) 2 = 9 2 a 2 + b 2 + c 2 = 81 2 ( a b + a c + b c ) (a+b+c)^2=9^2\\ \implies a^2+b^2+c^2=81-2(ab+ac+bc) Plugging this in ( ) (*) ,we get: a 2 + b 2 + c 2 a b + a c + b c 81 2 ( a b + a c + b c ) a b + a c + b c a b + a c + b c 27 a^2+b^2+c^2≥ ab+ac+bc\\ \implies 81-2(ab+ac+bc)≥ ab+ac+bc\\ \implies \boxed{ab+ac+bc≤27} Equality occurs as a = b = c = 3 a=b=c=3

Chew-Seong Cheong
May 23, 2016

Relevant wiki: Cauchy-Schwarz Inequality

Using Cauchy-Schwarz inequality , we have:

( a b + b c + c a ) 2 ( a 2 + b 2 + c 2 ) ( a 2 + b 2 + c 2 ) a b + b c + c a a 2 + b 2 + c 2 Equality occurs when a = b = c = 3 a b + b c + c a ( a + b + c ) 2 2 ( a b + b c + c a ) 3 ( a b + b c + c a ) ( a + b + c ) 2 a b + b c + c a 1 3 ( a + b + c ) 2 = 81 3 = 27 \begin{aligned} (ab+bc+ca)^2 & \le (a^2+b^2+c^2)(a^2+b^2+c^2) \\ \implies ab+bc+ca & \le a^2+b^2+c^2 \quad \quad \quad \quad \quad \quad \quad \quad \small \color{#3D99F6}{\text{Equality occurs when }a=b=c=3} \\ ab+bc+ca & \le (a+b+c)^2 - 2(ab+bc+ca) \\ 3(ab+bc+ca) & \le (a+b+c)^2 \\ ab+bc+ca & \le \frac{1}{3}(a+b+c)^2 = \frac{81}{3} = \boxed{27} \end{aligned}

0 pending reports

×

Problem Loading...

Note Loading...

Set Loading...